8
$\begingroup$

I have a city model (all one mesh) and a separate curved base plane. I would like to have the bottom faces of the each building in the city mesh flat against the plane and keep each building at 90 degrees to follow the curve of the base plane. This is illustrated here in the inception movie poster although this looks more like a sharper curve than I thinking but is a very good visual.


(source: wikimedia.org)

I have tried the shrink wrap modifier but cant seem to get the settings to keep each building 90 degrees to the curve. I cant use the particle method because I want to use a real city model that is one mesh.

How can I do the Inception style city on a curved plane effect in blender?

$\endgroup$

1 Answer 1

9
$\begingroup$

There are a few ways to bend a city like that.

One would be to create a curve that controls the deformation, and use it on a curve modifier applied to your city mesh.

enter image description here

enter image description here

The walls of the buildings will be perpendicular to the curve (and they might even crash into each other...)

Other very different option would be to use a lattice to control the deformation:

If you have the city and a curved plane. enter image description here

Create a lattice that is olny one segment in height, line it up with the base of the city and dimension it to the with and depth of the city.

enter image description here

Select the city object, add a lattice deform modifier and choose the lattice you'd just created as the control object.

enter image description here

Select then the lattice, apply a shrinkwrap modifier and use the curved plane as target.

enter image description here

The lattice will deform to hug the plane, and the city will deform following the lattice. Note that in this case the top of the buildings will remain parallel to the curved plane but the walls will remain vertical on the Z axis and will not crash into each other this time. Maybe not quite what you're looking for, but another fun option to be aware of.

$\endgroup$

You must log in to answer this question.

Not the answer you're looking for? Browse other questions tagged .